Author Topic: Almost an ansible? entanglement without classical particles or EM Comms  (Read 60812 times)

Offline Stormbringer

  • Full Member
  • ****
  • Posts: 1340
  • Liked: 239
  • Likes Given: 92
http://phys.org/news/2015-03-quantum-scheme-states-transmitting-physical.html

Quote
What's key is that during the entire process, the photon did not enter the channel between sender and receiver: As long as the photon passes through the channel, it will be absorbed by the ensemble if the control atom is in the ground state – but if the atom is in the Rydberg state, the photon will be absorbed by the detector. In this way, an unknown quantum state can be transferred between two distant participants without any physical particles traveling between them.


It looks to me like it says you can send information without sending particles or energy the classical way. So far all quantum communications had to have a classical component and it meant that no information exchange could actually take place faster than FTL. But this seems to say that is not needed.

But it also looks like they take pains to say (at least twice) that it applies to unknown information or an unknown state... So i am confused about whether this is FTL communications or not. What are they saying?

« Last Edit: 03/11/2015 02:28 am by Stormbringer »
When antigravity is outlawed only outlaws will have antigravity.

Offline Robotbeat

  • Senior Member
  • *****
  • Posts: 39270
  • Minnesota
  • Liked: 25240
  • Likes Given: 12115
Nope, you still can't send classical information in this manner faster than light.
Chris  Whoever loves correction loves knowledge, but he who hates reproof is stupid.

To the maximum extent practicable, the Federal Government shall plan missions to accommodate the space transportation services capabilities of United States commercial providers. US law http://goo.gl/YZYNt0

Offline JPLeRouzic

  • Member
  • Posts: 27
  • France, Rennes
  • Liked: 9
  • Likes Given: 29
http://phys.org/news/2015-03-quantum-scheme-states-transmitting-physical.html

Quote
What's key is that during the entire process, the photon did not enter the channel between sender and receiver: As long as the photon passes through the channel, it will be absorbed by the ensemble if the control atom is in the ground state – but if the atom is in the Rydberg state, the photon will be absorbed by the detector. In this way, an unknown quantum state can be transferred between two distant participants without any physical particles traveling between them.


It looks to me like it says you can send information without sending particles or energy the classical way. So far all quantum communications had to have a classical component and it meant that no information exchange could actually take place faster than FTL. But this seems to say that is not needed.

But it also looks like they take pains to say (at least twice) that it applies to unknown information or an unknown state... So i am confused about whether this is FTL communications or not. What are they saying?
My understanding (and I am not a scientist) is that a photon goes several times from Alice's side to Bob's side as it is shown in the pictures. So it is not FTL per definition.
I think what they meant is that contrary to quantum entanglement they don't need to transport states from the place of entanglement to the measurement place.
* In quantum entanglement at the moment one side makes a measurement, it can deduce the measurement at the other side, without looking at the other side but having painstakingly transported in a classical manner both entangled states from the place of entanglement to their present locations.
* Here the quantum state on Bob's side is determined by Bob influence on the atoms but this state information is not transported "classically" on the Alice's side. However photons are going from Alice's side to Bob' side and vice versa. The authors argue that as the photons don't really read the state on Bob's side, then the wavefunction doesn't collapse at the first travel and state information is transfered on Alice side without time being spent.

To give an analogy:
- in normal entanglement, one breaks a set of cards in two and transport them in two different locations. In one of them Alice browse her own subset and therefore knows instantly what Bob's subset is composed of.
- In the article, Bob breaks a set of cards in two and authorizes Alice to read classically in one of the two subsets, but only one card at a time. At the end Alice knows what is in the first subset and instantly knows what is in the second subset.
What Shakespeare said about EMDrive or blackholes is also valid for entanglement "much ado for nothing".
 8)
« Last Edit: 03/11/2015 09:02 am by JPLeRouzic »

Offline Stormbringer

  • Full Member
  • ****
  • Posts: 1340
  • Liked: 239
  • Likes Given: 92
When antigravity is outlawed only outlaws will have antigravity.

Offline KelvinZero

  • Senior Member
  • *****
  • Posts: 4286
  • Liked: 887
  • Likes Given: 201
The real problem is not how to build an instantaneous communicator. The problem is that two events at different locations that are simultaneous to one observer are not in general simultaneous to another, and you could trivially create a time paradox.


Offline Paul451

  • Senior Member
  • *****
  • Posts: 3553
  • Australia
  • Liked: 2518
  • Likes Given: 2180
This has always bugged me. Wouldn't the time paradox not matter, because the second observer could not communicate any information about the time paradox he witnessed back to the first observer before the paradox happens in the first observer's frame of reference?

Apparently there's always a frame of reference that would allow this. No matter how the FTL system worked, there's a way to design a paradox around it.

[I can occasionally get my head around specific cases, like communication using wormholes, but I don't understand the general case. You might have better luck.]

Offline ddunham

  • Member
  • Posts: 55
  • Liked: 5
  • Likes Given: 6
Wouldn't the time paradox not matter, because the second observer could not communicate any information about the time paradox he witnessed back to the first observer before the paradox happens in the first observer's frame of reference?

Just because it's not a problem in one frame doesn't keep it from being a problem in another frame. 

If you keep speeds below c, then no matter how you shift things around, all observers agree on causality.  Either A happened before B, B happened before A (both time like), or A and B are not connected (space like).  All inertial frames get the same result.

As soon as information can exceed c, then causality of some events changes in the view of some observers.  They will not agree on whether A caused B or B caused A, something that was well-defined before.  Because of this you can get a set of observers that see sequentially that A causes B, B causes C, and C causes A.  That's a problem.

Offline Bob Shaw

  • Full Member
  • ****
  • Posts: 1427
  • Liked: 727
  • Likes Given: 676
Information cannot exceed C; wavefronts may do so, but can't carry information.

We're trying to use English here to describe subtle mathematics, and our innate perception of causality etc interferes with any understanding of the outer universe - we are simply one of the Great Apes, looking out into a Universe which doesn't fit in with our mental predispositions, and attempting to erect a narrative.

Go and read Jastrow.

Offline QuantumG

  • Senior Member
  • *****
  • Posts: 9238
  • Australia
  • Liked: 4477
  • Likes Given: 1108
As soon as information can exceed c, then causality of some events changes in the view of some observers.  They will not agree on whether A caused B or B caused A, something that was well-defined before.  Because of this you can get a set of observers that see sequentially that A causes B, B causes C, and C causes A.  That's a problem.

Why is that a problem?

Who cares if two observers disagree about what they see? That happens all the time now and we all continue to exist.
Human spaceflight is basically just LARPing now.

Offline Nilof

  • Full Member
  • ****
  • Posts: 1177
  • Liked: 597
  • Likes Given: 707
As soon as information can exceed c, then causality of some events changes in the view of some observers.  They will not agree on whether A caused B or B caused A, something that was well-defined before.  Because of this you can get a set of observers that see sequentially that A causes B, B causes C, and C causes A.  That's a problem.

Why is that a problem?

Who cares if two observers disagree about what they see? That happens all the time now and we all continue to exist.

Because if you have an ansible that can transmit information faster than light, you can send information back into your past light cone, something that all observers will agree on, and you get things like grandfather paradoxes.
« Last Edit: 04/04/2015 01:00 pm by Nilof »
For a variable Isp spacecraft running at constant power and constant acceleration, the mass ratio is linear in delta-v.   Δv = ve0(MR-1). Or equivalently: Δv = vef PMF. Also, this is energy-optimal for a fixed delta-v and mass ratio.

Offline QuantumG

  • Senior Member
  • *****
  • Posts: 9238
  • Australia
  • Liked: 4477
  • Likes Given: 1108
Because if you have an ansible that can transmit information faster than light, you can send information back into your past light cone, something that all observers will agree on, and you get things like grandfather paradoxes.

Again.. who cares? While I think we can all agree that it would be strange, it's hardly much stranger than, say, Schrödinger's cat and - last I checked - there's little to support the notion that quantum mechanics should be rejected because it's so weird and wonderful. All the arguments against the possibility of faster than light travel, instantaneous communications or, heck, even time travel, amount to this same argument. We're supposed to cross our arms and scream "impossible!" because it'd ruin our neat little ideas of how causality works. Well boo hoo. We've known for a very long time that the real world is weirder than we can imagine.
Human spaceflight is basically just LARPing now.

Offline Nilof

  • Full Member
  • ****
  • Posts: 1177
  • Liked: 597
  • Likes Given: 707
Sure. But it is useful to point out that the consequences of even the slightest form of FTL effect are very drastic - it'd imply that the universe would be strongly acausal. For example, if you could use entanglement to transmit information, you could mess with events in billion-year old faraway stars or galaxies just by looking at them.

At some point the consequences become so strong that you'd have to start wondering why we haven't seen any time travellers from the future.
« Last Edit: 04/04/2015 02:51 pm by Nilof »
For a variable Isp spacecraft running at constant power and constant acceleration, the mass ratio is linear in delta-v.   Δv = ve0(MR-1). Or equivalently: Δv = vef PMF. Also, this is energy-optimal for a fixed delta-v and mass ratio.

Offline Star One

  • Senior Member
  • *****
  • Posts: 13996
  • UK
  • Liked: 3974
  • Likes Given: 220

Sure. But it is useful to point out that the consequences of even the slightest form of FTL effect are very drastic - it'd imply that the universe would be strongly acausal. For example, if you could use entanglement to transmit information, you could mess with events in billion-year old faraway stars or galaxies just by looking at them.

At some point the consequences become so strong that you'd have to start wondering why we haven't seen any time travellers from the future.

How would we know that we haven't, if you're intent on going down that particular metaphorical rabbit hole.

Offline KelvinZero

  • Senior Member
  • *****
  • Posts: 4286
  • Liked: 887
  • Likes Given: 201
Sure. But it is useful to point out that the consequences of even the slightest form of FTL effect are very drastic -
I think you might be confusing them if you say "sure". The implication is that FTL gives you timetravel and we can just get used to that. What it really gives you is a paradox. FTL is probably no more a route to time travel than 1+1=3 is a route to free energy.

Anyone who thinks they can live with it should at least attempt to decipher this new world and explain it to us. What sort of world is it where one observer at one velocity sees a man dying from a poisoned kool-aid while his phone rings beside him, while another observer at a different velocity sees him answer his phone and pour out the poison?

Offline MichaelBlackbourn

  • Full Member
  • *
  • Posts: 186
  • Liked: 305
  • Likes Given: 0
Sure. But it is useful to point out that the consequences of even the slightest form of FTL effect are very drastic -

Anyone who thinks they can live with it should at least attempt to decipher this new world and explain it to us. What sort of world is it where one observer at one velocity sees a man dying from a poisoned kool-aid while his phone rings beside him, while another observer at a different velocity sees him answer his phone and pour out the poison?

Manyworlds interpretation would.

Offline KelvinZero

  • Senior Member
  • *****
  • Posts: 4286
  • Liked: 887
  • Likes Given: 201
Manyworlds interpretation would.
I don't see how in this particular case. The two observers at different velocities could still come back together and compare results. In fact they could both meet up at the crime scene.

Offline Stormbringer

  • Full Member
  • ****
  • Posts: 1340
  • Liked: 239
  • Likes Given: 92
I think we are in trouble if our scientific analysis consists of "This violates some fudged principle created to make physicists feel better about the universe on philosophical grounds."

There is absolutely no evidence of which I am aware that any of these postulates about preserving order have any basis in physical facts let alone that they are universally true.

Furthermore physicists will evoke time travel when convenient. Feynman and wheeler posited advanced waves and retarded waves traveling into the future and the past to solve certain otherwise intractible problems.

Recent entanglement experiments strongly supported future measurements affecting the state of particles that had already ceased to exist.

Relativity allows (so far) limited time travel but it is time travel. all of it allowed by the rules; given a traversible wormhole which is not yet ruled out. Relativity allows it; physicists have *faith* however that something in Quantum Gravity theory which hasn't been discovered yet will forbid that.

*Faith* that some how "theory X" will forbid it. LOL. Better get out the robes, incense and ritual fetishes.

EDIT: Unless i am mistaken any relativistic travel allows time travel and we have observed physical objects with mass that are traveling at relativistic speed with our astronomical instruments. Certain celestial objects that got booted by a massive gravitational partner at some point. If one of these objects were to return to it's starting point at speed it would amount to time travel. 
« Last Edit: 04/05/2015 09:15 am by Stormbringer »
When antigravity is outlawed only outlaws will have antigravity.

Offline Nilof

  • Full Member
  • ****
  • Posts: 1177
  • Liked: 597
  • Likes Given: 707
Furthermore physicists will evoke time travel when convenient. Feynman and wheeler posited advanced waves and retarded waves traveling into the future and the past to solve certain otherwise intractible problems.


The Feynman propagator does indeed have a part that can seem to violate causality, but the existence of antimatter automatically ensures that these terms always exactly cancelled out so that all local observables commute(are independent) if they are separated by a spacelike distance.

What one really needs to appreciate is that for every form of physics discovered so far has not just been causal, but explicitly checking whether a theory is causal has been a very useful tool to better understand it.

First of all, any form of time-travel would have to rely on entirely new and undiscovered physics. There are theorems in both General Relativity and quantum field theory that show that the theories are causal under very general assumptions (CPT invariance, averaged null-energy condition).


With that said, you could split up the possibillities into five classes, in increasing order of unlikeliness:

1) "Boring physics" - Faster than light travel and information transfer is not possible. Causality holds.
2) Hawking's chronology protection conjecture - FTL possible, but no causality violation: Limited forms of faster than light travel including some types of wormholes are possible, but closed timelike curves are explicitly prohibited.
3 & 4) No causality but reasonable spacetimes: Time travel is possible, and paradoxes are handled by either a foliation of spacetime, or through some prescription of Novikov's self-consistency principle. In some viewpoints (usually many-worlds inspired) the two may be equivalent.
5) The universe is not a Hausdorff topological space. Really pedantic and difficult to construct any natural examples of, but occasionally included in such lists as it is a common axiom in GR theorems that all manifolds are Hausdorff.
« Last Edit: 04/05/2015 01:55 pm by Nilof »
For a variable Isp spacecraft running at constant power and constant acceleration, the mass ratio is linear in delta-v.   Δv = ve0(MR-1). Or equivalently: Δv = vef PMF. Also, this is energy-optimal for a fixed delta-v and mass ratio.

Offline Stormbringer

  • Full Member
  • ****
  • Posts: 1340
  • Liked: 239
  • Likes Given: 92
Even time travel where you could not effect your own timeline would be very very useful for information gathering and for stealing acquiring artifacts (to preserve them for posterity) and grabbing specimens say for botany, biology or zoology. if there are  "many worlds" then some would be so close to identical that the only difference could be some detail so minor as to be unnoticeable.
When antigravity is outlawed only outlaws will have antigravity.

Offline Paul451

  • Senior Member
  • *****
  • Posts: 3553
  • Australia
  • Liked: 2518
  • Likes Given: 2180
I think we are in trouble if our scientific analysis consists of "This violates some fudged principle created to make physicists feel better about the universe on philosophical grounds."
There is absolutely no evidence of which I am aware that any of these postulates about preserving order have any basis in physical facts let alone that they are universally true.

Like conservation of energy, etc, these core assumptions are tested every time we test a theory which has it as an assumption.

More so, every time someone comes up with a mathematical trick that appears to allow violation of causality, speed of light, conservations of energy/charge/etc, it always turns out that the effect can't be used to do so. Every single frickin' time. At some point you have to accept what the universe is telling you.

EDIT: Unless i am mistaken any relativistic travel allows time travel

But only forwards. There's no way for sub-FTL relativistic travel to result in travel back in time.

Offline QuantumG

  • Senior Member
  • *****
  • Posts: 9238
  • Australia
  • Liked: 4477
  • Likes Given: 1108
More so, every time someone comes up with a mathematical trick that appears to allow violation of causality, speed of light, conservations of energy/charge/etc, it always turns out that the effect can't be used to do so. Every single frickin' time. At some point you have to accept what the universe is telling you.

Ahh.. the feeling of every generation that they've learnt all there is to learn.
Human spaceflight is basically just LARPing now.

Offline KelvinZero

  • Senior Member
  • *****
  • Posts: 4286
  • Liked: 887
  • Likes Given: 201
The point should not be taken that instantaneous communication is impossible. The important point is that it isn't even defined what it means. Of course blah blah blah may be possible but that conversation can't hold my interest very long.

Someone mentioned many worlds before and I dismissed it but I might not have understood what they meant. I did actually come up with a many worlds-like idea a while ago. You could be communicating with alternate possibilities. For example suppose you had an infinitely powerful super computer at your location that sucked in every possible piece of information available at light speed and used this to extrapolate a possible future at a distant location, arbitrarily defined as 'now'. It would still be limited. It could not predict a cosmic-ray particle travelling towards you near lightspeed for example. If your ansible communication contained this sort of uncertainty then it would not violate relativity and more importantly we could consider what we actually had.

Offline Stormbringer

  • Full Member
  • ****
  • Posts: 1340
  • Liked: 239
  • Likes Given: 92
I feel this might be slightly related so I'll put it here and then give my reasoning:

http://phys.org/news/2015-04-unparticles-path-superconductivity.html

and a slightly different particle thing but i have my reasons:

http://phys.org/news/2015-04-supercomputer-lambda-baryon-quark-molecule.html

Now, if i understand the first article properly the component particles of an unparticle actually have mass individually but that when acting together they are massless. That's   ...interesting. So by putting massive particles together in the right configuration they act as if they are massless? Massless things can go really fast.

I would not mind some commentary on that.

Now the second article: a lambda Baryon is a regular baryon with a kaon appended to  the nucleon. Kaons normally go poof in a tiny tiny fraction of a second or something like that. This includes Kaonium pairs. But Kaons are stable if appended to other particles? or not? I think they may be. Does anyone see if it gives a decay time for lambda baryons?

Anyway if they are stable, in this case inside a baryon, does that mean that if there is an aggregation of them they might also be stable in and of themselves? A neutron is only stable in a nucleus unless they are squeezed together in a neutron star. A nucleus composed only of kaons or similar particles would be tinier than a regular atom and have stronger nuclear and electronic bonding.

When antigravity is outlawed only outlaws will have antigravity.

Offline JasonAW3

  • Senior Member
  • *****
  • Posts: 2443
  • Claremore, Ok.
  • Liked: 410
  • Likes Given: 14
What gets me is simple;  If a communications could be transmitted in real timeto another person lightyears away, with no lag time, (Other than the speed of sound going from the sender to the transmitter and then from the receiver to the person listening, how would that be a violation of Relativity?

So long as the reception of the message cannot arrive before it is received, where is the paradox?

     If data can be shared by entangled particles, again, so long as simultunaity is maintained between two points, where is the paradox?  (Yeah, it's a word I made up, I think, it means for an event to happen at two remote locations at the same time, as compared to each location's frame of reference).

     Mind you, this could not be applicable to objects moving at relativistic velocities, as the frame of reference would be massively skewed.  While data transmitted to an object moving at relativistic velocities would be vastly spead up, data transmitted from an object moving at relativistic velocities should be slowed down, relative to the temporal difference in reference frames.
My God!  It's full of universes!

Offline Mark K

  • Full Member
  • *
  • Posts: 137
  • Wisconsin
  • Liked: 78
  • Likes Given: 29
What gets me is simple;  If a communications could be transmitted in real time to another person light years away, with no lag time, (Other than the speed of sound going from the sender to the transmitter and then from the receiver to the person listening, how would that be a violation of Relativity?

So long as the reception of the message cannot arrive before it is (sent? added here) received, where is the paradox?


 You cannot send a message faster than light from one location to another that has space like curve between them without some (indefinite maybe infinite number of) frames of reference having the receipt occur before the sending. This is s standard problem of superluminal signalling in Special Relativity.

It is unavoidable from the way space and time are defined in S.R. I am pretty sure it was Robert Forward who had several articles that gave relatively humorous details of several of these situations, but I can't find any references now.

Offline Nilof

  • Full Member
  • ****
  • Posts: 1177
  • Liked: 597
  • Likes Given: 707
What gets me is simple;  If a communications could be transmitted in real timeto another person lightyears away, with no lag time, (Other than the speed of sound going from the sender to the transmitter and then from the receiver to the person listening, how would that be a violation of Relativity?

So long as the reception of the message cannot arrive before it is received, where is the paradox?

     If data can be shared by entangled particles, again, so long as simultunaity is maintained between two points, where is the paradox?  (Yeah, it's a word I made up, I think, it means for an event to happen at two remote locations at the same time, as compared to each location's frame of reference).

     Mind you, this could not be applicable to objects moving at relativistic velocities, as the frame of reference would be massively skewed.  While data transmitted to an object moving at relativistic velocities would be vastly spead up, data transmitted from an object moving at relativistic velocities should be slowed down, relative to the temporal difference in reference frames.

The violation is in the "no lag time" part, which is ill-defined because of relativity of simultaneity. If there is no special frame of reference, you would be able to do it in a different frame, where "before" and "after" is not the same as in the first frame.

Furthermore, there is the more fundamental fact that in any frame of reference, the time coordinate is a human convention(much like say, the z-coordinate in any spatial coordinate system) and only the proper time is a truly "natural" quantity. In general relativity it is a fundamentally flawed way to define simultaneity because time dilation happens not just because of speed but also because of gravitational gradients. This is why the GPS system needs to incorporate general relativity so that your phone won't tell you that your position is 15 km below Denver.

The only truly natural way to "sync up" the two ends that would be well-defined even in GR is to require that they are created in the same point, and have the ends synced up by the proper time that passes along their worldlines from the point where they were created. In that case you could construct something like the twins paradox where you could send one end on a spaceship back and forth at relativistic speeds and have one slow down relative to the other to construct a time machine.

This kind of synchronization is what happens to wormholes. In this particular case, there are possible mechanisms to prevent acausality, such as Hawking's chronology protection conjecture which states that wormholes in an acausal setup automatically collapse preventing time travel. That is, assuming wormholes actually exist.
« Last Edit: 04/08/2015 02:49 pm by Nilof »
For a variable Isp spacecraft running at constant power and constant acceleration, the mass ratio is linear in delta-v.   Δv = ve0(MR-1). Or equivalently: Δv = vef PMF. Also, this is energy-optimal for a fixed delta-v and mass ratio.

Offline Stormbringer

  • Full Member
  • ****
  • Posts: 1340
  • Liked: 239
  • Likes Given: 92
The funny thing is even if Hawking's *conjecture* is true; it does not prevent all time travel. Not by itself anyway.

The following is what people such as Dr John Kramer, Dr Woodward, Paul Davies are on record as having said about wormholes, time travel/causality issues related to wormholes, Cosmic back Reaction and Dr Hawking's conjecture:

 If you can get a wormhole set up between here and a distal destination you can travel into the future and (in a limited but still interesting and useful way) into the past. You just cannot go to a time before the wormhole was set up. Also; the feedback mechanisms intended to prevent any traversible wormhole at all actually end up providing the way to set up such a wormhole. Cosmic Back Reaction provides a means to enlarge, propel and steer wormholes when it was intended to collapse the wormhole before any naughty messing about with time travel could occur. CBR does not kick into full wormhole killing gear unless you set up a wormhole network in such a way that it could be used to travel into the past beyond the point when the wormhole was activated.
« Last Edit: 04/08/2015 12:00 am by Stormbringer »
When antigravity is outlawed only outlaws will have antigravity.

Offline RonM

  • Senior Member
  • *****
  • Posts: 3340
  • Atlanta, Georgia USA
  • Liked: 2231
  • Likes Given: 1584
I understand relativity of simultaneity and the statement that there is not a special frame of reference. However, what would constitute a special frame of reference? If it is something that can be measured from any frame of reference, then isn't the cosmic microwave background leftover from the Big Bang a special frame of reference? If I am missing something here, please enlighten me.

Offline ChrisWilson68

  • Senior Member
  • *****
  • Posts: 5266
  • Sunnyvale, CA
  • Liked: 4992
  • Likes Given: 6459
I think we are in trouble if our scientific analysis consists of "This violates some fudged principle created to make physicists feel better about the universe on philosophical grounds."

Fortunately, causality isn't "some fudged principle created to make physicists feel better about the universe".

The only people fudging here to make themselves feel better about the universe are those who want to believe faster-than-light travel or communication is possible in spite of all the very strong evidence that it is not.

Sure, we could find new evidence one day that calls into question whether FTL travel or communication is possible.  So far, there's been no such evidence.

There is absolutely no evidence of which I am aware that any of these postulates about preserving order have any basis in physical facts let alone that they are universally true.

Every experiment, everywhere, has supported the idea that causality has a basis in physical fact.  Everyday life experience also provides such evidence.  I can't think of anything at all that has more evidence going for it than causality.

Furthermore physicists will evoke time travel when convenient. Feynman and wheeler posited advanced waves and retarded waves traveling into the future and the past to solve certain otherwise intractible problems.

You're misunderstanding what they were talking about.  Those theories were explicitly consistent with the inability to communicate classical information or travel faster than light.

Recent entanglement experiments strongly supported future measurements affecting the state of particles that had already ceased to exist.

And that has absolutely nothing to do with violating causality.

Relativity allows (so far) limited time travel but it is time travel. all of it allowed by the rules; given a traversible wormhole which is not yet ruled out.

There are lots of things that are consistent with relativity but inconsistent with other evidence.  Relativity only describes part of known physics.

Relativity allows it; physicists have *faith* however that something in Quantum Gravity theory which hasn't been discovered yet will forbid that.

That's utter nonsense.  Physicists follow the evidence.  Causality is overwhelmingly supported by the evidence we currently have.  That's why physicists coming up with theories try to find theories that also support causality -- because a physical theory that matches the evidence is better than one that doesn't match the evidence.

*Faith* that some how "theory X" will forbid it. LOL. Better get out the robes, incense and ritual fetishes.

EDIT: Unless i am mistaken any relativistic travel allows time travel and we have observed physical objects with mass that are traveling at relativistic speed with our astronomical instruments. Certain celestial objects that got booted by a massive gravitational partner at some point. If one of these objects were to return to it's starting point at speed it would amount to time travel.

No, it wouldn't amount to time travel.  You seem to misunderstand the word "relativitstic" or its implications.  Going faster than light would amount to time travel.  The word "relativistic" means, roughly, "close to but less than the speed of light".
« Last Edit: 04/08/2015 05:31 am by ChrisWilson68 »

Offline ChrisWilson68

  • Senior Member
  • *****
  • Posts: 5266
  • Sunnyvale, CA
  • Liked: 4992
  • Likes Given: 6459
I understand relativity of simultaneity and the statement that there is not a special frame of reference. However, what would constitute a special frame of reference? If it is something that can be measured from any frame of reference, then isn't the cosmic microwave background leftover from the Big Bang a special frame of reference? If I am missing something here, please enlighten me.

A "special frame of reference" would mean your communication system only works in some special frame of reference, not in all the other frames of reference.  So far, all evidence says there is no special frame of reference, meaning laws of physics all work in all frames of reference.

Lets say you have a device that allows you "instant" communication from point A to point B.  "instant" in what frame of reference?  If relativity is right and physics is the same in all frames of reference, then your communication device should work in any frame of reference, and it should allow communication both from point A to point B and from point B to point A.

Events that are simultaneous in one frame of reference are not simultaneous in another frame of reference if they happen at different points in space.  If the points are 1 light hour apart in frame F1, events that are up to one hour apart at points A and B in frame F1 will be simultaneous in some other frame F2.  So, if you can communicate simultaneously in frame F2, you can send a message from point A to point B that is half an hour in the past in F1 but simultaneous in frame F2.  Then, send the message back one minute later from point B to point A using instant communication in F1 and it arrives at point A 29 minutes before it was sent from point A.

Offline Nilof

  • Full Member
  • ****
  • Posts: 1177
  • Liked: 597
  • Likes Given: 707
I understand relativity of simultaneity and the statement that there is not a special frame of reference. However, what would constitute a special frame of reference? If it is something that can be measured from any frame of reference, then isn't the cosmic microwave background leftover from the Big Bang a special frame of reference? If I am missing something here, please enlighten me.

In the more general sense of general relativity, a reference frame is just any set of coordinates describing spacetime, which are allowed to be curvilinear or to describe an accelerating or rotating background.

Since coordinates are just a human convention, general relativity is invariant under general coordinate transformations. Effectively this means that trying to define a "global time" naturally is doomed to failure because nature doesn't care about whether humans have an easy time describing it.

If there are no closed timelike curves, you can introduce a partial time ordering: if there is a timelike geodesic from A to B, A objectively happens before B. But events with a spacelike separation will generally be causally unrelated and you cannot say whether one or the other happens first.
« Last Edit: 04/08/2015 12:04 pm by Nilof »
For a variable Isp spacecraft running at constant power and constant acceleration, the mass ratio is linear in delta-v.   Δv = ve0(MR-1). Or equivalently: Δv = vef PMF. Also, this is energy-optimal for a fixed delta-v and mass ratio.

Offline Stormbringer

  • Full Member
  • ****
  • Posts: 1340
  • Liked: 239
  • Likes Given: 92
I think we are in trouble if our scientific analysis consists of "This violates some fudged principle created to make physicists feel better about the universe on philosophical grounds."

Fortunately, causality isn't "some fudged principle created to make physicists feel better about the universe".

The only people fudging here to make themselves feel better about the universe are those who want to believe faster-than-light travel or communication is possible in spite of all the very strong evidence that it is not.

Sure, we could find new evidence one day that calls into question whether FTL travel or communication is possible.  So far, there's been no such evidence.

There is absolutely no evidence of which I am aware that any of these postulates about preserving order have any basis in physical facts let alone that they are universally true.

Every experiment, everywhere, has supported the idea that causality has a basis in physical fact.  Everyday life experience also provides such evidence.  I can't think of anything at all that has more evidence going for it than causality.

Furthermore physicists will evoke time travel when convenient. Feynman and wheeler posited advanced waves and retarded waves traveling into the future and the past to solve certain otherwise intractible problems.

You're misunderstanding what they were talking about.  Those theories were explicitly consistent with the inability to communicate classical information or travel faster than light.

Recent entanglement experiments strongly supported future measurements affecting the state of particles that had already ceased to exist.

And that has absolutely nothing to do with violating causality.

Relativity allows (so far) limited time travel but it is time travel. all of it allowed by the rules; given a traversible wormhole which is not yet ruled out.

There are lots of things that are consistent with relativity but inconsistent with other evidence.  Relativity only describes part of known physics.

Relativity allows it; physicists have *faith* however that something in Quantum Gravity theory which hasn't been discovered yet will forbid that.

That's utter nonsense.  Physicists follow the evidence.  Causality is overwhelmingly supported by the evidence we currently have.  That's why physicists coming up with theories try to find theories that also support causality -- because a physical theory that matches the evidence is better than one that doesn't match the evidence.

*Faith* that some how "theory X" will forbid it. LOL. Better get out the robes, incense and ritual fetishes.

EDIT: Unless i am mistaken any relativistic travel allows time travel and we have observed physical objects with mass that are traveling at relativistic speed with our astronomical instruments. Certain celestial objects that got booted by a massive gravitational partner at some point. If one of these objects were to return to it's starting point at speed it would amount to time travel.

No, it wouldn't amount to time travel.  You seem to misunderstand the word "relativitstic" or its implications.  Going faster than light would amount to time travel.  The word "relativistic" means, roughly, "close to but less than the speed of light".
No- i know well what these terms mean. And merely relativistic travel is also time travel due to gamma factor. if i go to a distant star at 99.99 percent C i travel in time. I experience an abbreviated time compared to observers at home. in my perspective the trip takes mere moments or weeks at worst. That is time travel. It's not your definition of time travel; but it is time travel.

If i return home at 99.99 percent light speed; I will have aged moments or weeks at worst while everyone I greet at journey's end will be about 9 or ten years older. I will have effectively traveled to the future skipping all the inconvenient things in between moments.
When antigravity is outlawed only outlaws will have antigravity.

Offline RonM

  • Senior Member
  • *****
  • Posts: 3340
  • Atlanta, Georgia USA
  • Liked: 2231
  • Likes Given: 1584
Thanks ChrisWilson68 and Nilof for your replies on my special frame of reference question. Now I understand it.

Offline JasonAW3

  • Senior Member
  • *****
  • Posts: 2443
  • Claremore, Ok.
  • Liked: 410
  • Likes Given: 14
Information cannot exceed C; wavefronts may do so, but can't carry information.

We're trying to use English here to describe subtle mathematics, and our innate perception of causality etc interferes with any understanding of the outer universe - we are simply one of the Great Apes, looking out into a Universe which doesn't fit in with our mental predispositions, and attempting to erect a narrative.

Go and read Jastrow.

Explain quantum tunneling then.

     A particle going from point A to point B without transitioning through the interveining space certainly seems to violate the concept of information traveling faster than light.

     Admittedly this, so far, has only been observed on the nanoscopic scale, but the transition certainly SEEMS to be happining at faster than light velocities.
My God!  It's full of universes!

Offline gospacex

  • Senior Member
  • *****
  • Posts: 3024
  • Liked: 543
  • Likes Given: 604
I think we are in trouble if our scientific analysis consists of "This violates some fudged principle created to make physicists feel better about the universe on philosophical grounds."

It's not just philosophical grounds. Allowing time travel would mean that if you travel back in time and try to kill yourself, there will be a force, distinct from normal physical laws, which will not let you do that. You try to shoot yourself, but your gun doesn't fire. Or bullet swerves away. I can see why this is seen as absurd. Local physics can't possibly depend on whether you came from the future or not.

Still, people are not censoring FTL theories. It is okay to research them; but any theory which postulates that FTL is possible needs to explain how paradoxes are avoided.

Offline QuantumG

  • Senior Member
  • *****
  • Posts: 9238
  • Australia
  • Liked: 4477
  • Likes Given: 1108
I love the way causality is sacred but universality of time is whatever. We are products of our era. Both relativity and quantum mechanics (and, heck, the accelerating expansion of the universe and just about everything else in modern physics) are so adverse to common sense that we should not be surprised if causality is the next casualty.
 
Human spaceflight is basically just LARPing now.

Offline Nilof

  • Full Member
  • ****
  • Posts: 1177
  • Liked: 597
  • Likes Given: 707
There is nothing special about requiring extraordinary claims to have extraordinary evidence. Self-consistent non-causal universes have been studied to great lengths and it certainly isn't "taboo" to mention them.

They are just spectacularly different from everything we have observed so far and the existence of a CTC explicitly kills off most of thermodynamics and information theory. For a computer next to a CTC you don't just have P = NP, you in fact have something even stronger than P = PSPACE as all computation becomes memory limited only.

Information cannot exceed C; wavefronts may do so, but can't carry information.

We're trying to use English here to describe subtle mathematics, and our innate perception of causality etc interferes with any understanding of the outer universe - we are simply one of the Great Apes, looking out into a Universe which doesn't fit in with our mental predispositions, and attempting to erect a narrative.

Go and read Jastrow.

Explain quantum tunneling then.

     A particle going from point A to point B without transitioning through the interveining space certainly seems to violate the concept of information traveling faster than light.

     Admittedly this, so far, has only been observed on the nanoscopic scale, but the transition certainly SEEMS to be happining at faster than light velocities.

Tunneling does not lead to any kind of FTL travel. Anything that has the Dirac equation or quantum electrodynamics at the bottom will trivially be causal. Any other result is a result of miscalculation or the improper use of some approximation.

There are lots of phenomena that can make it look like you are moving something faster than the speed of light in physics, but as you explain what is happening in terms of more fundamental physics it turns out that you can't ever transmit any information faster than light with it and that the apparently "FTL" component is allowed.

I like to use constructions in cellular automata like as an analogy. In that example, it looks like an LWSS has been moved faster than the "speed of light" in Conway's game of life, but when you accept that all patterns in cellular automata are just a collection of cells it becomes obvious that you can't actually transmit information that way. Similarly, any patterns in physics are just excitations in fields rather than what we intuitively think of as "particles" and the rules that govern these trump any illusions you may encounter at a higher level of abstraction.
« Last Edit: 04/08/2015 10:31 pm by Nilof »
For a variable Isp spacecraft running at constant power and constant acceleration, the mass ratio is linear in delta-v.   Δv = ve0(MR-1). Or equivalently: Δv = vef PMF. Also, this is energy-optimal for a fixed delta-v and mass ratio.

Offline Stormbringer

  • Full Member
  • ****
  • Posts: 1340
  • Liked: 239
  • Likes Given: 92
I have a question on unparticles related to Classical electrodynamics, QED and the Standard Model.

My understanding of an unparticle is that it is an aggregate of positive mass particles which acting as a group sums it's mass to zero.

In the above mentioned frameworks as i understand them normal massed particles sum to a slightly positive mass but the sources side of the equation included a large negative mass component or else divergence cannot be eliminated. In the above frame works the various source contributors to electron mass such as magnetic moment dipole moment angular velocity self gravity and so forth overcome the negative bare mass with a slight over balance turning particle mass positive.

well then you have the unparticle where the positive mass is overcame by just enough something to sum out to zero. So what is it? Negative bare mass enlarged? positive contributor reduced? additional negative energy above the component level in the field section of the equations? what mechanism allows for the zero mass property? because you know... this is not supposed to happen.

and if you can sum positive masses to zero mass what stops you from summing to a negative mass with some other configuration in the unparticle?

 
When antigravity is outlawed only outlaws will have antigravity.

Offline KelvinZero

  • Senior Member
  • *****
  • Posts: 4286
  • Liked: 887
  • Likes Given: 201
Explain quantum tunneling then.

     A particle going from point A to point B without transitioning through the interveining space certainly seems to violate the concept of information traveling faster than light.

     Admittedly this, so far, has only been observed on the nanoscopic scale, but the transition certainly SEEMS to be happining at faster than light velocities.
I sure can't explain quantum tunnelling :)

..but the physicists who claim to understand it say that the mathematics that predict quantum tunnelling also explain why it cannot be used to transfer information faster than light.

Sure, that is sort of like an appeal to authority. But you really have to accept both statements together. Either "There is this thing called quantum tunnelling but it does not let information travel faster than light, because that is what the mathematics says", or "Commonly accepted quantum mechanics is analysed by thousands of doofuses with PhDs and implemented by thousands of doofus engineers who just can't spot the obvious thing we can spot without much experience with quantum mechanics." :)

Offline ChrisWilson68

  • Senior Member
  • *****
  • Posts: 5266
  • Sunnyvale, CA
  • Liked: 4992
  • Likes Given: 6459
EDIT: Unless i am mistaken any relativistic travel allows time travel and we have observed physical objects with mass that are traveling at relativistic speed with our astronomical instruments. Certain celestial objects that got booted by a massive gravitational partner at some point. If one of these objects were to return to it's starting point at speed it would amount to time travel.

No, it wouldn't amount to time travel.  You seem to misunderstand the word "relativitstic" or its implications.  Going faster than light would amount to time travel.  The word "relativistic" means, roughly, "close to but less than the speed of light".
No- i know well what these terms mean. And merely relativistic travel is also time travel due to gamma factor. if i go to a distant star at 99.99 percent C i travel in time. I experience an abbreviated time compared to observers at home. in my perspective the trip takes mere moments or weeks at worst. That is time travel. It's not your definition of time travel; but it is time travel.

If i return home at 99.99 percent light speed; I will have aged moments or weeks at worst while everyone I greet at journey's end will be about 9 or ten years older. I will have effectively traveled to the future skipping all the inconvenient things in between moments.

OK, then you're just making up your own definition for a term that already has a very widely understood meaning.  What's the point of that other than deliberate miscommunication?

Offline Stormbringer

  • Full Member
  • ****
  • Posts: 1340
  • Liked: 239
  • Likes Given: 92
EDIT: Unless i am mistaken any relativistic travel allows time travel and we have observed physical objects with mass that are traveling at relativistic speed with our astronomical instruments. Certain celestial objects that got booted by a massive gravitational partner at some point. If one of these objects were to return to it's starting point at speed it would amount to time travel.

No, it wouldn't amount to time travel.  You seem to misunderstand the word "relativitstic" or its implications.  Going faster than light would amount to time travel.  The word "relativistic" means, roughly, "close to but less than the speed of light".
No- i know well what these terms mean. And merely relativistic travel is also time travel due to gamma factor. if i go to a distant star at 99.99 percent C i travel in time. I experience an abbreviated time compared to observers at home. in my perspective the trip takes mere moments or weeks at worst. That is time travel. It's not your definition of time travel; but it is time travel.

If i return home at 99.99 percent light speed; I will have aged moments or weeks at worst while everyone I greet at journey's end will be about 9 or ten years older. I will have effectively traveled to the future skipping all the inconvenient things in between moments.

OK, then you're just making up your own definition for a term that already has a very widely understood meaning.  What's the point of that other than deliberate miscommunication?
No. I am not making up a new definition of time travel. Other people, some of them physicists have called the scenario I described time travel. It's in the popular literature.
When antigravity is outlawed only outlaws will have antigravity.

Offline Stormbringer

  • Full Member
  • ****
  • Posts: 1340
  • Liked: 239
  • Likes Given: 92
no the light cones -the time lines diverge. If i am in the crew of that ship my time is abbreviated (radically) compared to stationary observers back home. and as I said I am not alone in calling this situation time travel.
When antigravity is outlawed only outlaws will have antigravity.

Offline Bob Shaw

  • Full Member
  • ****
  • Posts: 1427
  • Liked: 727
  • Likes Given: 676
Even time travel where you could not effect your own timeline would be very very useful for information gathering and for stealing acquiring artifacts (to preserve them for posterity) and grabbing specimens say for botany, biology or zoology. if there are  "many worlds" then some would be so close to identical that the only difference could be some detail so minor as to be unnoticeable.

Tell that to the dinosaurs who invented the Iridium Bomb.

Offline Patchouli

  • Senior Member
  • *****
  • Posts: 4490
  • Liked: 253
  • Likes Given: 457
I hope one day we will find a way to send information faster then c as the speed of light lag is an inconvenience even within the confines of the solar system.
Though the existence of some sorta FTL com may explain the Fermi paradox with SETI as maybe civilizations only use radio or laser communications for a short period before moving to something else.
« Last Edit: 04/19/2015 01:21 am by Patchouli »

Offline blast335

  • Member
  • Posts: 3
  • The alien human
  • ohio
  • Liked: 0
  • Likes Given: 0
I had this idea too, so how it work's in my mind is that they have 2 entangled sets, one for receiving and one for sending, they would come up with a code, like Morse code, and send photons into their sending particle, then on the other side, the entangled particle would emit a photon and that photon would activate a detector, then a computer attached to that detector would decode the message. Assuming I under stand all of the details.

Offline JasonAW3

  • Senior Member
  • *****
  • Posts: 2443
  • Claremore, Ok.
  • Liked: 410
  • Likes Given: 14
I've always wondered if Quantum Tunneling, outside of a gravity well, could effectively produce simultanious communication over interstellar distances.
My God!  It's full of universes!

Offline SteveD

  • Full Member
  • ***
  • Posts: 312
  • United States
  • Liked: 83
  • Likes Given: 10
I have the strangest hunch that everyone here is wrong.  You can send a message back in time with a quantum effect, but that message cannot be distinguished from noise (decoded or decrypted) until such a time that it would not violate causality.  For example: entangle a photon.  Travel one light year distant.  You send a message.  Your recipient gets it now.  The recipient then sends back a reply that goes on "the stack" a year in the past.  You cannot distinguish the contents of the message you received a year ago, or even that a message has been sent, until now (or most likely a couple of weeks from now when whatever decryption equipment you have finishes processing the signal).  Because distances on one planet are short, it is quicker to receive a cheat sheet via EM broadcast than do the decryption.

Offline Stormbringer

  • Full Member
  • ****
  • Posts: 1340
  • Liked: 239
  • Likes Given: 92
not ansible related but topical:

http://phys.org/news/2015-08-physicist-unveils-entangling-massive.html

its a proposal to demonstrate quantum entanglement of 100 gram masses; the largest massive objects so far tried.
When antigravity is outlawed only outlaws will have antigravity.

Offline ChrisWilson68

  • Senior Member
  • *****
  • Posts: 5266
  • Sunnyvale, CA
  • Liked: 4992
  • Likes Given: 6459
I had this idea too, so how it work's in my mind is that they have 2 entangled sets, one for receiving and one for sending, they would come up with a code, like Morse code, and send photons into their sending particle, then on the other side, the entangled particle would emit a photon and that photon would activate a detector, then a computer attached to that detector would decode the message. Assuming I under stand all of the details.

How many times does this same myth have to be busted?  Entanglement does not give faster than light communication.

This is a well-known result in physics.

Base your fantasies of faster-than-light communication on something else.  Entanglement won't do it for you.

Offline ChrisWilson68

  • Senior Member
  • *****
  • Posts: 5266
  • Sunnyvale, CA
  • Liked: 4992
  • Likes Given: 6459
I've always wondered if Quantum Tunneling, outside of a gravity well, could effectively produce simultanious communication over interstellar distances.

Nope.

Quantum tunneling is a well-understood part of quantum physics.  The rules under which it happens have been proven mathematically to never transmit information faster than light.

You really do need new physics to get faster-than-light communication.  There's no point in trying to use phenomena that are described already by known physics.

Offline blast335

  • Member
  • Posts: 3
  • The alien human
  • ohio
  • Liked: 0
  • Likes Given: 0
I had this idea too, so how it work's in my mind is that they have 2 entangled sets, one for receiving and one for sending, they would come up with a code, like Morse code, and send photons into their sending particle, then on the other side, the entangled particle would emit a photon and that photon would activate a detector, then a computer attached to that detector would decode the message. Assuming I under stand all of the details.

How many times does this same myth have to be busted?  Entanglement does not give faster than light communication.

This is a well-known result in physics.

Base your fantasies of faster-than-light communication on something else.  Entanglement won't do it for you.
Well we can teleport quantum states through entangled particles, so why can't we devise a system to interpret these teleported states into messages?

Offline QuantumG

  • Senior Member
  • *****
  • Posts: 9238
  • Australia
  • Liked: 4477
  • Likes Given: 1108
Well we can teleport quantum states through entangled particles, so why can't we devise a system to interpret these teleported states into messages?

What do you think they're called quantum states and not quantum messages?

Human spaceflight is basically just LARPing now.

Offline ChrisWilson68

  • Senior Member
  • *****
  • Posts: 5266
  • Sunnyvale, CA
  • Liked: 4992
  • Likes Given: 6459
I had this idea too, so how it work's in my mind is that they have 2 entangled sets, one for receiving and one for sending, they would come up with a code, like Morse code, and send photons into their sending particle, then on the other side, the entangled particle would emit a photon and that photon would activate a detector, then a computer attached to that detector would decode the message. Assuming I under stand all of the details.

How many times does this same myth have to be busted?  Entanglement does not give faster than light communication.

This is a well-known result in physics.

Base your fantasies of faster-than-light communication on something else.  Entanglement won't do it for you.
Well we can teleport quantum states through entangled particles, so why can't we devise a system to interpret these teleported states into messages?

There's a theorem in quantum physics called the "No Communication Theorem".  It says that you can't use entanglement for communication.  The theorem has been mathematically proven from the rules of quantum physics.

https://en.wikipedia.org/wiki/No-communication_theorem

Offline Stormbringer

  • Full Member
  • ****
  • Posts: 1340
  • Liked: 239
  • Likes Given: 92
When antigravity is outlawed only outlaws will have antigravity.

Offline ChrisWilson68

  • Senior Member
  • *****
  • Posts: 5266
  • Sunnyvale, CA
  • Liked: 4992
  • Likes Given: 6459
a (2 Bit) ansible!

http://news.sciencemag.org/physics/2015/12/physicists-figure-out-how-retrieve-information-black-hole

No, not at all.  The article has nothing to do with faster-than-light communication.  It's about using entanglement (which is known not to enable faster-than-light communication) and how it relates to information not being lost in a black hole.

Offline 1

  • Full Member
  • ***
  • Posts: 367
  • El Segundo, CA
  • Liked: 749
  • Likes Given: 10
Explain quantum tunneling then.

     A particle going from point A to point B without transitioning through the interveining space certainly seems to violate the concept of information traveling faster than light.

     Admittedly this, so far, has only been observed on the nanoscopic scale, but the transition certainly SEEMS to be happining at faster than light velocities.
I sure can't explain quantum tunnelling :)

..but the physicists who claim to understand it say that the mathematics that predict quantum tunnelling also explain why it cannot be used to transfer information faster than light.


I can. The issue with tunneling isn't that a particle travels from region A to region B in zero time (the word 'point' has subtle connotations and should not be used), it's that said particle appears to traverse a region that's classically disallowed.

This is a consequence of the QM hamiltonian equation requiring fields 'n' wave solutions. In the classical world, having a particle bounce off a wall (an effective reflection coefficient of 1) is a perfectly acceptable boundary condition; and a particle measured in region A will forever be bound to region A. When dealing with fields and waves, the only time this is mathematically possible is when you have a barrier of infinite potential energy; and barriers of infinite potential energy simply haven't been found nor are likely to be found anywhere in nature. Consequently, a quantum field (read: the particles' wavefunction) requires solutions to maintain smooth-'n'-continuous boundary conditions similar to those required by just about every other known field. This requires a non-zero permeation into an otherwise forbidden zone, and is analogous to evanescent solutions found in classical E&M. These solutions decay very, very, very rapidly, but never actually reach zero magnitude. Your reflection coefficient is always less than 1. Thus, at the end of the forbidden zone, region B, a small but nonzero part of the wavefunction couples back into an allowed state. This allows a particle once found in region A to appear in region B.

The direct answer to your question about tunneling, however, is this. There is still a very lively ongoing debate about proper interpretation of what it means to both collapse and re-expand the wavefunction, but the long and the short of it is that a wavefuction, once collapsed (read: a particles position is first measured) then needs time to re-expand and permeate that forbidden region. Subsequent measurements on that particle, if done before the wavefunction has time to re-expand, will always measure the particle in region A. Only after a sufficient length of time has passed, which is greater than the amount of time it would take light to travel from region A to B, could the particle then be found to have tunneled into the other region. The speed limit of C is not an explicit requirement of quantum mechanics as I understand it, but it is a requirement that appears to be obeyed in experiment.

Offline Stormbringer

  • Full Member
  • ****
  • Posts: 1340
  • Liked: 239
  • Likes Given: 92
When antigravity is outlawed only outlaws will have antigravity.

Offline Stormbringer

  • Full Member
  • ****
  • Posts: 1340
  • Liked: 239
  • Likes Given: 92
When antigravity is outlawed only outlaws will have antigravity.

Offline RotoSequence

  • Senior Member
  • *****
  • Posts: 2208
  • Liked: 2068
  • Likes Given: 1535
https://phys.org/news/2018-02-two-way-quantum-particle.html

is it an ansible yet?

Almost certainly not, but I don't understand why it isn't. I found this related paper on Arxiv: https://arxiv.org/abs/1802.05102

Quote from: Abstract
Experimental two-way communication with one photon

Superposition of two or more states is one of the fundamental concepts of quantum mechanics and provides the basis for several advantages quantum information processing offers. In this work, we experimentally demonstrate that quantum superposition permits two-way communication between two distant parties that can exchange only one particle once, an impossible task in classical physics. This is achieved by preparing a single photon in a coherent superposition of the two parties' locations. Furthermore, we show that this concept allows the parties to perform secure quantum communication, where the transmitted bits and even the direction of communication remain private. These important features can lead to the development of new quantum communication schemes, which are simultaneously secure and resource-efficient.

« Last Edit: 02/27/2018 06:14 am by RotoSequence »

Offline ppnl

  • Full Member
  • ***
  • Posts: 312
  • Liked: 208
  • Likes Given: 19


Well just looking at the diagram it still shows a time gap from t=0 to t=tau ruled by the speed of light. The only thing they did is make it possible to send information both ways but at the cost of a device half way between.

Classically quantum information looks like it needs to travel faster than light in order to do what it does. But in the end you have to have a real particle travel the distance in order to read that quantum message. There is a mathematical proof that the rules of QM do not allow faster than light communication. Looking for such a a thing is akin to looking for a way to square the circle with only compass and straightedge. It just ain't gonna happen.

That does not mean there is a proof that FTL communication cannot happen although such a thing is a PITA theoretically. For example it would allow sending information back in time. But it does mean that you will have to move past quantum mechanics because the tools furnished by QM are not sufficient.

Offline meberbs

  • Senior Member
  • *****
  • Posts: 3096
  • Liked: 3379
  • Likes Given: 777
The speed limit of C is not an explicit requirement of quantum mechanics as I understand it, but it is a requirement that appears to be obeyed in experiment.
2 year old post, but to expand on this, straight quantum mechanics based on the Schrodinger equation is non-relativistic, and has no protections for the speed of light.

As I understand it, the Dirac Equation (relativistic extension of quantum mechanics) is properly relativistic with full compatibility with special relativity, and the speed limit of c in inherent in it.

Offline ppnl

  • Full Member
  • ***
  • Posts: 312
  • Liked: 208
  • Likes Given: 19
The speed limit of C is not an explicit requirement of quantum mechanics as I understand it, but it is a requirement that appears to be obeyed in experiment.
2 year old post, but to expand on this, straight quantum mechanics based on the Schrodinger equation is non-relativistic, and has no protections for the speed of light.

As I understand it, the Dirac Equation (relativistic extension of quantum mechanics) is properly relativistic with full compatibility with special relativity, and the speed limit of c in inherent in it.

It is true that simple QM has no protection for the speed of light. But in order to use quantum entanglement to pass information there has to be a real particle that travels the distance. Whatever limits the speed of that particle also limits the speed of the information transferred by quantum entanglement. That limit would be set by relativity.

And yes relativistic QM has that built in.

In any case the article clearly shows a time interval from t=0 to t=tau that it takes to transmit the signal. There is no reason to speculate about an ansible.

Offline sghill

  • Full Member
  • ****
  • Posts: 1682
  • United States
  • Liked: 2092
  • Likes Given: 3200
The speed limit of C is not an explicit requirement of quantum mechanics as I understand it, but it is a requirement that appears to be obeyed in experiment.
2 year old post, but to expand on this, straight quantum mechanics based on the Schrodinger equation is non-relativistic, and has no protections for the speed of light.

As I understand it, the Dirac Equation (relativistic extension of quantum mechanics) is properly relativistic with full compatibility with special relativity, and the speed limit of c in inherent in it.

It is true that simple QM has no protection for the speed of light. But in order to use quantum entanglement to pass information there has to be a real particle that travels the distance. Whatever limits the speed of that particle also limits the speed of the information transferred by quantum entanglement. That limit would be set by relativity.

Yes. My limited understanding of entanglement is that you wind up with a Schrodinger's Cat situation until you transmit end-state information in real time back to the observer, which obviously removes the advantage of entanglement-enabled communications.
Bring the thunder!

Offline Stormbringer

  • Full Member
  • ****
  • Posts: 1340
  • Liked: 239
  • Likes Given: 92
I think this is at least half an ansible since the two stations could possibly halve the time to communicate regardless of C. So half an ansible?  ;)
When antigravity is outlawed only outlaws will have antigravity.

Offline ppnl

  • Full Member
  • ***
  • Posts: 312
  • Liked: 208
  • Likes Given: 19
The speed limit of C is not an explicit requirement of quantum mechanics as I understand it, but it is a requirement that appears to be obeyed in experiment.
2 year old post, but to expand on this, straight quantum mechanics based on the Schrodinger equation is non-relativistic, and has no protections for the speed of light.

As I understand it, the Dirac Equation (relativistic extension of quantum mechanics) is properly relativistic with full compatibility with special relativity, and the speed limit of c in inherent in it.

It is true that simple QM has no protection for the speed of light. But in order to use quantum entanglement to pass information there has to be a real particle that travels the distance. Whatever limits the speed of that particle also limits the speed of the information transferred by quantum entanglement. That limit would be set by relativity.

Yes. My limited understanding of entanglement is that you wind up with a Schrodinger's Cat situation until you transmit end-state information in real time back to the observer, which obviously removes the advantage of entanglement-enabled communications.

No, the limit is still just the speed of light. Otherwise you could send messages back in time and in principle get replys before you send a message. There can be no speed advantage at all or you break physics.

Think of it this way. Say you split the message into two halves neither of which can be read without the other. If one half can only go at the speed of light then no matter how fast the other half can go the message itself is limited to the speed of light. This is what makes quantum mechanics a local theory rather than a nonlocal theory. It has some strange features for a local theory but it is still local. 

Offline dustinthewind

  • Full Member
  • ****
  • Posts: 902
  • U.S. of A.
  • Liked: 313
  • Likes Given: 355
I wanted to suggest Leonard Susskind's video ER=EPR from this forum discussion here, http://physicshelpforum.com/philosophy-physics/12217-does-qm-contradict-relativity.html
and
https://en.wikipedia.org/wiki/ER%3DEPR




He links the concept of quantum entanglement to black holes which open the doors to a kind of a worm hole or Einstein–Rosen bridge.  It turns out quantum entangled particles may also be such a bridge on a micro scale.  It's kind of iffy because he is talking about pushing all that like charge into the form of a black hole (the wiki mentions charge trapped in a magnetic field) but what I came out of it with was that one black hole was anti matter entangled with the other which is matter. 

I was already questioning if anti-matter could be negative energy running backwards in time, making it appear as positive energy.  The reverse time stuff has connotations to Feynman's backward time propagators but in such a way as to mask their existence (we don't see the signal) but allow the universe to allow multiple possibilities (quantum) to exist simultaneously, till collapse, and instantly determine the results over any distance.  Maybe it could be thought of as a wormhole too. 

But then quantum entangled particles don't have to be matter and anti-matter. 

The black hole appeared to possibly have entangled space between them.  I attached an image of my speculation.  I was pondering what might happen if you create an e-p pair and annihilate the p with an e of another pair.  Would the remaining pair become entangled or would there be any teleportation?  Haven't watched the 2nd video yet. 
« Last Edit: 06/20/2018 12:22 am by dustinthewind »
Follow the science? What is science with out the truth.  If there is no truth in it it is not science.  Truth is found by open discussion and rehashing facts not those that moderate it to fit their agenda.  In the end the truth speaks for itself.  Beware the strong delusion and lies mentioned in 2ndThesalonians2:11.  The last stage of Babylon is transhumanism.  Clay mingled with iron (flesh mingled with machine).  MK ultra out of control.  Consider bill gates patent 202060606 (666), that hacks the humans to make their brains crunch C R Y P T O. Are humans hackable animals or are they protected like when Jesus cast out the legion?

Offline ppnl

  • Full Member
  • ***
  • Posts: 312
  • Liked: 208
  • Likes Given: 19
Well you really can't talk about matter black holes and antimatter black holes because black holes almost certainly violate baryon number. We don't have any black holes to play with but there are some very simple and very general arguments that make it extremely likely that black holes violate baryon number.

If ER = EPR is true then you can send a message to a distant black hole faster than light. But that message is stuck inside a black hole and you can't get it out.

If you let a positron and electron annihilate you would create a pair of photons. If you started with two entangled pairs you would still have four particles after the annihilation. What would be entangled? The simple answer is that the wave would collapse. The more complex answer is that the quantum information is spread over all four particles leading to decoherence. This kind of decoherence is how the macro world which appears to be classical is derived from the weird non-classical world of quantum mechanics.
« Last Edit: 06/24/2018 03:48 pm by ppnl »

Offline dustinthewind

  • Full Member
  • ****
  • Posts: 902
  • U.S. of A.
  • Liked: 313
  • Likes Given: 355
Had a moment of confusion.  It isn't anti-matter in its real state I think that provides reverse time communication but rather anti-matter in the vacuum state.  The annihilated form of it.  I think it may provide the reverse time communication to collapse states.  I think its that reverse time that provides the bridge that links quantum states. 

For instance light as a wave passes through two slits.  The wave pattern predicts the appearance of the photon but once the photon appears all the energy of the photon is concentrated at the point of absorption.  Its almost like the reverse time operator goes back in time and focuses the energy at that exact location.  Edit: the random distribution of photons by the waves state I believe is induced by the random thermal background of the vacuum superimposed over the waves state - this would be the hidden variable in a sense.

Creating electron-positron pairs in the vacuum in a quantum states of superposition.  Its like they were created out of the vacuum but the vacuum has not determined their exact orientation yet, upon interaction the vacuum uses the anti-matter in the vacuum state or the virtual stuff, to determine the final state of the other quantum entangled particle.  The wormhole bridge between the two. 

Tunneling for instance.  A particle bangs on a barrier and it has a vacuum wave associated with the particle so particle-wave hybrid.  Compound that the vacuum has inherent vacuum energy fluctuations if the particle's vacuum waves passes through the barrier then based on the quantum vacuum energy fluctuation there is a chance the far part of the wave will receive enough energy to re-create the particle on the other side of the barrier.  When the particle is created on the other side of the barrier it induces reverse time operators that annihilate the particles previous position via negative energy and the particle exists at its new position. 

I suspect that the nature of Quantum reality is fundamentally related to the vacuum and negative energy, reverse time operators.  Even Richard Feynman diagrams I have been told have reverse time operations. 

Causal pictures
The notion of going back in time is acausal, meaning it is excluded automatically in a Hamiltonian formulation. For this reason, it took a long time for this approach to be appreciated and accepted. Stueckelberg proposed this interpretation of antiparticles in the late 1930s, but Feynman's presentation made it stick.

In Feynman diagrams, the future is not determined from the past by stepping forward timestep by timestep, it is determined by tracing particle paths proper-time by proper-time. The diagram formalism therefore is philosophically very different from the Hamiltonian field theory formalism, so much so Feynman was somewhat disappointed that they were equivalent.

They are not as easily equivalent when you go to string theory, because string theory is an S-matrix theory formulated entirely in Feynman language, not in Hamiltonian language. The Hamiltonian formulation of strings requires a special slicing of space time, and even then, it is less clear and elegant than the Feynman formulation, which is just as acausal and strange. The strings backtrack in time just like particles do, since they reproduce point particles at infinite tension.

If you philosophically dislike acausal formalisms, you can say (in field theory) that the Hamiltonian formalism is fundamental, and that you believe in crossing and CPT, and then you don't have to talk about going back in time. Since crossing and CPT are the precise manifestations of the statement that antimatter is matter going back in time, you really aren't saying anything different, except philosophically. But the philosophy motivates crossing and CPT.

or

http://www.johnagowan.org/feynman.html

Another example is an electron in orbit around an atom.  As it falls in it encounters vacuum polarized electron positron pairs via the atoms electric field.  As it falls in it emits energy and it trapped around the atom in a wave state.  I think it exist mostly as a vacuum fluctuation.  The electron looks like a cloud because it is a cloud of vacuum fluctuation.  It isn't actually whizzing around the nucleus, but the vacuum wave is, so hence no radiation. 

Even radioactive decay.  If the distribution of proton and neutrons isn't right this may create points around the nucleus that have electric fields in excess increasing the chance via vacuum fluctuations of particle creation outside the nucleus?  Haven't really thought this last one through much but seems plausible. 

Continued:  Relativistic length contraction might be speculated to be a polarization wave in the vacuum where in front of a moving particle, negative energy particles that subtract time build up, while behind positive energy builds up adding time, causing relativistic time travel. 

General relativity might be speculated to be something similar, where matter polarizes the vacuum where negative energy reduces the local mass distributing it over space - a field.  the build up of negative energy near matter, causing lower altitude clocks to tick slower than those higher.  Possibly related to the nucleus of atoms polarizing the vacuum.
 
« Last Edit: 06/30/2018 03:32 pm by dustinthewind »
Follow the science? What is science with out the truth.  If there is no truth in it it is not science.  Truth is found by open discussion and rehashing facts not those that moderate it to fit their agenda.  In the end the truth speaks for itself.  Beware the strong delusion and lies mentioned in 2ndThesalonians2:11.  The last stage of Babylon is transhumanism.  Clay mingled with iron (flesh mingled with machine).  MK ultra out of control.  Consider bill gates patent 202060606 (666), that hacks the humans to make their brains crunch C R Y P T O. Are humans hackable animals or are they protected like when Jesus cast out the legion?

Offline Star-Drive

  • Member
  • Full Member
  • ****
  • Posts: 925
  • TX/USA
  • Liked: 1031
  • Likes Given: 31
Had a moment of confusion.  It isn't anti-matter in its real state I think that provides reverse time communication but rather anti-matter in the vacuum state.  The annihilated form of it.  I think it may provide the reverse time communication to collapse states.  I think its that reverse time that provides the bridge that links quantum states. 

For instance light as a wave passes through two slits.  The wave pattern predicts the appearance of the photon but once the photon appears all the energy of the photon is concentrated at the point of absorption.  Its almost like the reverse time operator goes back in time and focuses the energy at that exact location. 

Creating electron-positron pairs in the vacuum in a quantum states of superposition.  Its like they were created out of the vacuum but the vacuum has not determined their exact orientation yet, upon interaction the vacuum uses the anti-matter in the vacuum state or the virtual stuff, to determine the final state of the other quantum entangled particle.  The wormhole bridge between the two. 

Tunneling for instance.  A particle bangs on a barrier and it has a vacuum wave associated with the particle so particle-wave hybrid.  Compound that the vacuum has inherent vacuum energy fluctuations if the particle's vacuum waves passes through the barrier then based on the quantum vacuum energy fluctuation there is a chance the far part of the wave will receive enough energy to re-create the particle on the other side of the barrier.  When the particle is created on the other side of the barrier it induces reverse time operators that annihilate the particles previous position via negative energy and the particle exists at its new position. 

I suspect that the nature of Quantum reality is fundamentally related to the vacuum and negative energy, reverse time operators.  Even Richard Feynman diagrams I have been told have reverse time operations. 

Causal pictures
The notion of going back in time is acausal, meaning it is excluded automatically in a Hamiltonian formulation. For this reason, it took a long time for this approach to be appreciated and accepted. Stueckelberg proposed this interpretation of antiparticles in the late 1930s, but Feynman's presentation made it stick.

In Feynman diagrams, the future is not determined from the past by stepping forward timestep by timestep, it is determined by tracing particle paths proper-time by proper-time. The diagram formalism therefore is philosophically very different from the Hamiltonian field theory formalism, so much so Feynman was somewhat disappointed that they were equivalent.

They are not as easily equivalent when you go to string theory, because string theory is an S-matrix theory formulated entirely in Feynman language, not in Hamiltonian language. The Hamiltonian formulation of strings requires a special slicing of space time, and even then, it is less clear and elegant than the Feynman formulation, which is just as acausal and strange. The strings backtrack in time just like particles do, since they reproduce point particles at infinite tension.

If you philosophically dislike acausal formalisms, you can say (in field theory) that the Hamiltonian formalism is fundamental, and that you believe in crossing and CPT, and then you don't have to talk about going back in time. Since crossing and CPT are the precise manifestations of the statement that antimatter is matter going back in time, you really aren't saying anything different, except philosophically. But the philosophy motivates crossing and CPT.

or

http://www.johnagowan.org/feynman.html

Another example is an electron in orbit around an atom.  As it falls in it encounters vacuum polarized electron positron pairs via the atoms electric field.  As it falls in it emits energy and it trapped around the atom in a wave state.  I think it exist mostly as a vacuum fluctuation.  The electron looks like a cloud because it is a cloud of vacuum fluctuation.  It isn't actually whizzing around the nucleus, but the vacuum wave is, so hence no radiation. 

Even radioactive decay.  If the distribution of proton and neutrons isn't right this may create points around the nucleus that have electric fields in excess increasing the chance via vacuum fluctuations of particle creation outside the nucleus?  Haven't really thought this last one through much but seems plausible. 

Continued:  Relativistic length contraction might be speculated to be a polarization wave in the vacuum where in front of a moving particle, negative energy particles that subtract time build up, while behind positive energy builds up adding time, causing relativistic time travel. 

General relativity might be speculated to be something similar, where matter polarizes the vacuum where negative energy reduces the local mass distributing it over space - a field.  the build up of negative energy near matter, causing lower altitude clocks to tick slower than those higher.  Possibly related to the nucleus of atoms polarizing the vacuum.
 


All:

You might be interested on Dr. Harold (Sonny) Whites and his Eagleworks team's thoughts on this subject especially about what actually is going on when an electron is in orbit around a nucleus.
Star-Drive

Offline ChrisWilson68

  • Senior Member
  • *****
  • Posts: 5266
  • Sunnyvale, CA
  • Liked: 4992
  • Likes Given: 6459
Had a moment of confusion.  It isn't anti-matter in its real state I think that provides reverse time communication but rather anti-matter in the vacuum state.  The annihilated form of it.  I think it may provide the reverse time communication to collapse states.  I think its that reverse time that provides the bridge that links quantum states.

None of that makes a lick of sense.  Just forming sentences out of terms from physics in a different order than others isn't new physics.

Offline dustinthewind

  • Full Member
  • ****
  • Posts: 902
  • U.S. of A.
  • Liked: 313
  • Likes Given: 355
Had a moment of confusion.  It isn't anti-matter in its real state I think that provides reverse time communication but rather anti-matter in the vacuum state.  The annihilated form of it.  I think it may provide the reverse time communication to collapse states.  I think its that reverse time that provides the bridge that links quantum states.

None of that makes a lick of sense.  Just forming sentences out of terms from physics in a different order than others isn't new physics.

Let us speculate.  Say we have two non antimatter ions Quantum entangled and then separated over a large distance.  If anti-matter was the reverse time operator there would be no anti-matter involved.  however, there is anti-matter involved in the vacuum and that antimatter is never truly annihilated.  It can never reach an absolute zero temperature so what made up e-p pairs still osculate in the vacuum.  So when one quantum entangled state collapses the other state instantly collapses via help from the vacuum.

It is the same with photons through the double slit experiment.  The photon wave passes through both slit and touches all points on the screen.  Even though the wave touches some points on the screen first rather than other points later.  At some point the vacuum wave associated with the photon and vacuum thermal fluctuations finds just the right frequency to be absorbed.  When it is absorbed the signal can propagate back in time and instantly cancel the wave (Collapse of the quantum wave function).   The energy is then concentrated at that exact point as a photon.

Also with quantum tunneling negative energy swallows the previous particles position.  If I remember correctly Quantum tunneling appears to be instantaneous.

Bold areas edited for clarity
« Last Edit: 07/07/2018 02:06 am by dustinthewind »
Follow the science? What is science with out the truth.  If there is no truth in it it is not science.  Truth is found by open discussion and rehashing facts not those that moderate it to fit their agenda.  In the end the truth speaks for itself.  Beware the strong delusion and lies mentioned in 2ndThesalonians2:11.  The last stage of Babylon is transhumanism.  Clay mingled with iron (flesh mingled with machine).  MK ultra out of control.  Consider bill gates patent 202060606 (666), that hacks the humans to make their brains crunch C R Y P T O. Are humans hackable animals or are they protected like when Jesus cast out the legion?

Offline meberbs

  • Senior Member
  • *****
  • Posts: 3096
  • Liked: 3379
  • Likes Given: 777
Let us speculate.  Say we have two non antimatter ions Quantum entangled and then separated over a large distance.  If anti-matter was the reverse time operator there would be no anti-matter.  however, there is anti-matter in the vacuum and that antimatter is never truly annihilated because it can never reach an absolute zero temperature.  So when one state collapses the other state instantly collapses.

It is the same with photons through the double slit experiment.  The photon wave passes through both slit and touches all points on the screen.  Even though the wave touches some points on the screen first rather than other points later.  At some point the vacuum wave associated with the photon and vacuum thermal fluctuations finds just the right frequency to be absorbed.  When it is absorbed the signal can propagate back in time and instantly cancel the wave.   The energy is then concentrated at that exact point as a photon.

Also with quantum tunneling negative energy swallows the previous particles position.  If I remember correctly Quantum tunneling appears to be instantaneous.
Your sentences still don't mean anything. There are multiple problems, but one repeated problem is your use of the word "instantly/intantaneous." That word is literally undefined when talking about relativity (It can be defined, but not in an absolute sense like you are using it.) Also, other than containing the word "anti-matter" there seems to be no connection between any of the sentences in your first paragraph.

Offline dustinthewind

  • Full Member
  • ****
  • Posts: 902
  • U.S. of A.
  • Liked: 313
  • Likes Given: 355
Let us speculate.  Say we have two non antimatter ions Quantum entangled and then separated over a large distance.  If anti-matter was the reverse time operator there would be no anti-matter.  however, there is anti-matter in the vacuum and that antimatter is never truly annihilated because it can never reach an absolute zero temperature.  So when one state collapses the other state instantly collapses.

It is the same with photons through the double slit experiment.  The photon wave passes through both slit and touches all points on the screen.  Even though the wave touches some points on the screen first rather than other points later.  At some point the vacuum wave associated with the photon and vacuum thermal fluctuations finds just the right frequency to be absorbed.  When it is absorbed the signal can propagate back in time and instantly cancel the wave.   The energy is then concentrated at that exact point as a photon.

Also with quantum tunneling negative energy swallows the previous particles position.  If I remember correctly Quantum tunneling appears to be instantaneous.
Your sentences still don't mean anything. There are multiple problems, but one repeated problem is your use of the word "instantly/intantaneous." That word is literally undefined when talking about relativity (It can be defined, but not in an absolute sense like you are using it.) Also, other than containing the word "anti-matter" there seems to be no connection between any of the sentences in your first paragraph.

That is exactly why Einstein called it spooky action-at-a-distance. It was spooky because it was faster than light or instantaneous. 

einstein-spooky-action-starlight-quantum-entanglement

Hence, the negative energy Wheeler Feynman reverse time operators to allow such a thing to occur.  Possibly worm holes so to speak.
« Last Edit: 07/07/2018 01:58 am by dustinthewind »
Follow the science? What is science with out the truth.  If there is no truth in it it is not science.  Truth is found by open discussion and rehashing facts not those that moderate it to fit their agenda.  In the end the truth speaks for itself.  Beware the strong delusion and lies mentioned in 2ndThesalonians2:11.  The last stage of Babylon is transhumanism.  Clay mingled with iron (flesh mingled with machine).  MK ultra out of control.  Consider bill gates patent 202060606 (666), that hacks the humans to make their brains crunch C R Y P T O. Are humans hackable animals or are they protected like when Jesus cast out the legion?

Offline meberbs

  • Senior Member
  • *****
  • Posts: 3096
  • Liked: 3379
  • Likes Given: 777
Let us speculate.  Say we have two non antimatter ions Quantum entangled and then separated over a large distance.  If anti-matter was the reverse time operator there would be no anti-matter.  however, there is anti-matter in the vacuum and that antimatter is never truly annihilated because it can never reach an absolute zero temperature.  So when one state collapses the other state instantly collapses.

It is the same with photons through the double slit experiment.  The photon wave passes through both slit and touches all points on the screen.  Even though the wave touches some points on the screen first rather than other points later.  At some point the vacuum wave associated with the photon and vacuum thermal fluctuations finds just the right frequency to be absorbed.  When it is absorbed the signal can propagate back in time and instantly cancel the wave.   The energy is then concentrated at that exact point as a photon.

Also with quantum tunneling negative energy swallows the previous particles position.  If I remember correctly Quantum tunneling appears to be instantaneous.
Your sentences still don't mean anything. There are multiple problems, but one repeated problem is your use of the word "instantly/intantaneous." That word is literally undefined when talking about relativity (It can be defined, but not in an absolute sense like you are using it.) Also, other than containing the word "anti-matter" there seems to be no connection between any of the sentences in your first paragraph.

That is exactly why Einstein called it spooky action-at-a-distance. It was spooky because it was faster than light or instantaneous. 
Faster than light is always instantaneous to somebody, time travel to somebody else, and normal flow of time to a third person. When you don't define who you are talking about, then descriptions like the one you provided lose all meaning.

Offline dustinthewind

  • Full Member
  • ****
  • Posts: 902
  • U.S. of A.
  • Liked: 313
  • Likes Given: 355
Had a moment of confusion.  It isn't anti-matter in its real state I think that provides reverse time communication but rather anti-matter in the vacuum state.  The annihilated form of it.  I think it may provide the reverse time communication to collapse states.  I think its that reverse time that provides the bridge that links quantum states. 

For instance light as a wave passes through two slits.  The wave pattern predicts the appearance of the photon but once the photon appears all the energy of the photon is concentrated at the point of absorption.  Its almost like the reverse time operator goes back in time and focuses the energy at that exact location. 

Creating electron-positron pairs in the vacuum in a quantum states of superposition.  Its like they were created out of the vacuum but the vacuum has not determined their exact orientation yet, upon interaction the vacuum uses the anti-matter in the vacuum state or the virtual stuff, to determine the final state of the other quantum entangled particle.  The wormhole bridge between the two. 

Tunneling for instance.  A particle bangs on a barrier and it has a vacuum wave associated with the particle so particle-wave hybrid.  Compound that the vacuum has inherent vacuum energy fluctuations if the particle's vacuum waves passes through the barrier then based on the quantum vacuum energy fluctuation there is a chance the far part of the wave will receive enough energy to re-create the particle on the other side of the barrier.  When the particle is created on the other side of the barrier it induces reverse time operators that annihilate the particles previous position via negative energy and the particle exists at its new position. 

I suspect that the nature of Quantum reality is fundamentally related to the vacuum and negative energy, reverse time operators.  Even Richard Feynman diagrams I have been told have reverse time operations. 

Causal pictures
The notion of going back in time is acausal, meaning it is excluded automatically in a Hamiltonian formulation. For this reason, it took a long time for this approach to be appreciated and accepted. Stueckelberg proposed this interpretation of antiparticles in the late 1930s, but Feynman's presentation made it stick.

In Feynman diagrams, the future is not determined from the past by stepping forward timestep by timestep, it is determined by tracing particle paths proper-time by proper-time. The diagram formalism therefore is philosophically very different from the Hamiltonian field theory formalism, so much so Feynman was somewhat disappointed that they were equivalent.

They are not as easily equivalent when you go to string theory, because string theory is an S-matrix theory formulated entirely in Feynman language, not in Hamiltonian language. The Hamiltonian formulation of strings requires a special slicing of space time, and even then, it is less clear and elegant than the Feynman formulation, which is just as acausal and strange. The strings backtrack in time just like particles do, since they reproduce point particles at infinite tension.

If you philosophically dislike acausal formalisms, you can say (in field theory) that the Hamiltonian formalism is fundamental, and that you believe in crossing and CPT, and then you don't have to talk about going back in time. Since crossing and CPT are the precise manifestations of the statement that antimatter is matter going back in time, you really aren't saying anything different, except philosophically. But the philosophy motivates crossing and CPT.

or

http://www.johnagowan.org/feynman.html

Another example is an electron in orbit around an atom.  As it falls in it encounters vacuum polarized electron positron pairs via the atoms electric field.  As it falls in it emits energy and it trapped around the atom in a wave state.  I think it exist mostly as a vacuum fluctuation.  The electron looks like a cloud because it is a cloud of vacuum fluctuation.  It isn't actually whizzing around the nucleus, but the vacuum wave is, so hence no radiation. 

Even radioactive decay.  If the distribution of proton and neutrons isn't right this may create points around the nucleus that have electric fields in excess increasing the chance via vacuum fluctuations of particle creation outside the nucleus?  Haven't really thought this last one through much but seems plausible. 

Continued:  Relativistic length contraction might be speculated to be a polarization wave in the vacuum where in front of a moving particle, negative energy particles that subtract time build up, while behind positive energy builds up adding time, causing relativistic time travel. 

General relativity might be speculated to be something similar, where matter polarizes the vacuum where negative energy reduces the local mass distributing it over space - a field.  the build up of negative energy near matter, causing lower altitude clocks to tick slower than those higher.  Possibly related to the nucleus of atoms polarizing the vacuum.
 


All:

You might be interested on Dr. Harold (Sonny) Whites and his Eagleworks team's thoughts on this subject especially about what actually is going on when an electron is in orbit around a nucleus.

I wanted to tell you thanks for sharing these papers.  I enjoyed the concept.  I wanted to suggest that they can possibly extend the concept further.  I think the reason superconductors super-conduct is because a group of atoms share a vacuum orbital across the entire material when cooled to a certain extent.  There is no resistance to current because of a similar reason why atoms don't emit light continuously while the electron is in stable orbit.  The electron being in a non-unique state in the vacuum with acoustic vibrations so to speak is also what I think is happening in superconductors. 

As a result the few electrons that are involved in superconducting appear to pair up - the same as what happens in electron orbitals. 
« Last Edit: 07/13/2018 01:58 pm by dustinthewind »
Follow the science? What is science with out the truth.  If there is no truth in it it is not science.  Truth is found by open discussion and rehashing facts not those that moderate it to fit their agenda.  In the end the truth speaks for itself.  Beware the strong delusion and lies mentioned in 2ndThesalonians2:11.  The last stage of Babylon is transhumanism.  Clay mingled with iron (flesh mingled with machine).  MK ultra out of control.  Consider bill gates patent 202060606 (666), that hacks the humans to make their brains crunch C R Y P T O. Are humans hackable animals or are they protected like when Jesus cast out the legion?

Tags:
 

Advertisement NovaTech
Advertisement Northrop Grumman
Advertisement
Advertisement Margaritaville Beach Resort South Padre Island
Advertisement Brady Kenniston
Advertisement NextSpaceflight
Advertisement Nathan Barker Photography
1